Đến nội dung

Minhnksc nội dung

Có 110 mục bởi Minhnksc (Tìm giới hạn từ 05-05-2020)



Sắp theo                Sắp xếp  

#697572 ĐỀ THI OLYMPIC TOÁN TRƯỜNG ĐÔNG TOÁN PHỔ THÔNG KHU VỰC BẮC TRUNG BỘ NĂM 2017

Đã gửi bởi Minhnksc on 01-12-2017 - 20:16 trong Thi HSG cấp Tỉnh, Thành phố. Olympic 30-4. Đề thi và kiểm tra đội tuyển các cấp.

Bài 5: Số nguyên dương $k$ có liên quan gì đến bài toán ?

À; đoạn sau là giới thiệu định nghĩa về lũy thừa đúng là gì nên nó không liên quan đến bài toán đâu

P/s: mình thấy đoạn đấy hơi thừa




#697533 Tìm tất cả các bộ số $(m;n)$

Đã gửi bởi Minhnksc on 30-11-2017 - 22:44 trong Tổ hợp và rời rạc

Cho hình chữ nhật $m\times n$ ($m\ge 2;n\ge 2$).Trên hình chữ nhật có một số các số nguyên cho trước; ta thực hiện các thao tác lấy một hình chữ nhật $1\times x$ hoặc $x\times 1$ ($x\ge 2$) và tiếp tục cộng thêm $k$ vào hai ô vuông ở hai đầu của hình chữ nhật đó rồi cộng thêm $k+1$ vào các ô vuông nằm giữa 2 ô vuông trên (nếu có) với $k\in \mathbb{Z}\setminus \left\{0\right\}$. Hãy tìm tất cả các bộ $(m;n)$ thỏa mãn với bất kì các số nguyên cho trước; tồn tại dãy hữu hạn các thao tác như trên sao cho đến thao tác cuối cùng; ta nhận được các số nguyên bằng nhau trên mọi ô vuông.




#696987 Đề chọn HSG QG Trung Quốc 2018

Đã gửi bởi Minhnksc on 21-11-2017 - 23:33 trong Thi HSG Quốc gia và Quốc tế

Ngày thứ nhất

Bài 1. Cho số nguyên dương $n$. Gọi $A_n$ là tập các số nguyên tố $p$ sao cho tồn tại các số nguyên dương $a;b$ thỏa mãn $\frac{a+b}{p}$ và $\frac{a^n+b^n}{p}$ là các số nguyên nguyên tố cùng nhau với $p$. Nếu $A_n$ hữu hạn, gọi $f(n)$ là số phần tử của nó.
a) Chứng minh $A_n$ hữu hạn khi và chỉ khi $n\ne 2$.
b) Cho $m;k$ là các số nguyên dương lẻ và $d=(m,k)$. Chứng minh
$f(d)\le f(k)+f(m)-f(mk)\le 2f(d)$
Bài 2. Cho $n;k$ là các số nguyên dương và tập
$T=\left\{(x;y;z)\in \mathbb{N}^3|1\le x,y,z\le n\right\}$
Biết $3n^2-3n+1+k$ điểm của $T$ được tô đỏ sao cho nếu $P,Q$ là các điểm đỏ và $PQ$ song song với một trong các trục thì tất cả các điểm thuộc $PQ$ đều được tô đỏ. Chứng minh tồn tại ít nhất $k$ hình lập phương đơn vị mà tất cả các đỉnh của chúng đều mang màu đỏ
Bài 3. Cho $q$ là số nguyên dương không phải là lập phương của một số nguyên. Chứng minh tồn tại hằng số dương $C$ sao cho với mỗi số nguyên dương $n$, ta có $\left\{nq^{\frac{1}{3}}\right\}+\left\{nq^{\frac{2}{3}}\right\}\ge Cn^{-\frac{1}{2}}$

 

Ngày thứ hai

Bài 4. Cho $ABCD$ là một tứ giác nội tiếp với $P$ là giao điểm của hai đường chéo. $(ADP)$ cắt đoạn $AB$ tại $A$ và $E$. $(PBC)$ cắt đoạn $AB$ tại $B$ và $F$. Gọi $I,J$ lần lượt là tâm đường tròn nội tiếp của các tam giác $ADE$ và $BCF$. Các đoạn $IJ$ và $AC$ cắt nhau tại $K$. Chứng minh các điểm $A,I,K,E$ cùng nằm trên một đường tròn.
Bài 5. Cho $n\ge 3$ là một số lẻ và giả sử rằng mỗi ô của bảng ô vuông $n\times n$ đã được tô bởi một trong hai màu đen hoặc trắng. Hai ô vuông được gọi là kề nhau nếu chúng có cùng màu và chung một đỉnh. Hai ô vuông $a,b$ được gọi là liên thông nếu tồn tại một dãy ô $c_1;...;c_n$ sao cho $c_1=a;c_k=b$ và $c_i;c_{i+1}$ kề nhau với mọi $i=\overline{1,n}$. Tìm số $M$ lớn nhất sao cho tồn tại một cách tô màu có $M$ ô vuông đôi một liên thông.
Bài 6. Cho các số nguyên dương $n,k$ thỏa mãn $n>k$ và $a_;a_2;...a_n\in (k-1,k)$. Xét các số thực dương $x_1;x_2;...;x_n$ có tính chất: Với mỗi $\mathbb{I}\subseteq \left\{1;2;...;n\right\}$, $|\mathbb{I}|=k$, ta có $\sum_{i\in \mathbb{I}}x_i\le\sum_{i\in \mathbb{I}}a_i$ . Tìm giá trị lớn nhất của $x_1.x_2...x_n$

Nguồn: Nguyễn Trung Tuân 

https://nttuan.org/2...2018/#more-7198




#696971 Nghịch lý căn bậc chẵn

Đã gửi bởi Minhnksc on 21-11-2017 - 21:28 trong Nghịch lý

Bạn không hiểu à  :(  Ý mình là nếu $\sqrt[n]{a}$ với n chẵn thì luôn có hai căn bậc hai, vây phải viết như nào mới đúng? $\sqrt[n]{a}$ = x và $\sqrt[n]{a}$ = -x hay -$\sqrt[n]{a}$ = -x; $\sqrt[n]{a}$ = x? Nếu cách viết thứ hai như vậy thì có nhất thiết phải đặt ra khái niệm căn bậc hai số học? Vả lại nếu cách viết 1 đúng thì x = -x?

ý mình là mình đang bàn về cái cbh số học kia :)




#696951 Nghịch lý căn bậc chẵn

Đã gửi bởi Minhnksc on 21-11-2017 - 18:36 trong Nghịch lý

Vậy cách viết đúng phải là -$\sqrt[n]{a}$ = -x hả bạn?

Vậy sao người ta còn đặt ra căn bậc hai số học làm chi rối rắm?

CBH số học là căn mà kết quả của nó là số dương; người ta sinh ra như vậy bởi lẽ số thực dương $A$ bất kì luôn có 2 căn bậc chẵn; thế cứ kí hiệu $\sqrt[n]{a}$ thế này thì biết đâu mà lần ???




#696854 Bài kiểm tra số 2 trường Đông Toán Học miền Nam.

Đã gửi bởi Minhnksc on 19-11-2017 - 22:14 trong Thi HSG cấp Tỉnh, Thành phố. Olympic 30-4. Đề thi và kiểm tra đội tuyển các cấp.

Bài 7:

b) Xét số tự nhiên $N$ sao cho $N=k_{1}k_{2}...k_{n}$

Khi đó số các số tự nhiên nhỏ hơn $N$ là nghiệm của pt $N\equiv x_{i}(mod k_{i})$(ta kí hiệu là $y_{i}$) sẽ không lớn hơn $\frac{N}{k_{i}}$ 

Mà $y_{1}+y_{2}+...+y_{n}\ge N$ (thật vậy nếu $y_{1}+y_{2}+...+y_{n}\le N$ thì sẽ tồn tại một số tự nhiên nhỏ hơn $N$ không thỏa mãn đề)

do đó $\frac{N}{k_{1}}+\frac{N}{k_{2}}+...+\frac{N}{k_{n}}\ge N$ 

Từ đó ta có $\sum_{i=1}^{n}\frac{1}{k_{i}}\ge 1$




#696245 Chứng minh rằng các vật thể luôn có các điểm chung nằm trong một hình phẳng c...

Đã gửi bởi Minhnksc on 08-11-2017 - 21:11 trong Dãy số - Giới hạn

Mở rộng bài trên lên thành hai chiều bằng cách thay các đoạn bằng các hình chữ nhật có cạnh song song với trục tọa độ.

Rồi mở rộng thêm tí nữa là các hình hộp có các cạnh song song trục tọa độ.

Sau đó, với mỗi khối đa diện đã cho, ta "bọc" bằng một hình hộp nhỏ nhất có thể. Chứng minh các hình hộp này cũng lồng nhau. Xong áp dụng cái mở rộng vừa trên là sẽ có kết quả bài toán.

Sau khi sửa lại đề thì ý tưởng của bài toán này cũng gần giống như thế ạ. 

Bài này em chế ra nhưng không để ý cái $V\rightarrow 0$ dẫn đến chuỗi vô hạn các vật thể đã cho không chỉ có một điểm chung duy nhất.

Nếu vật thể đã cho co lại về mọi phía để có $lim_{n\rightarrow +\infty}V_n=0$ thì một điểm chung duy nhất có lẽ là đúng. Nhưng chỉ co lại về một phía nào đó thì lại không đúng




#696078 Chứng minh rằng các vật thể luôn có các điểm chung nằm trong một hình phẳng c...

Đã gửi bởi Minhnksc on 04-11-2017 - 21:38 trong Dãy số - Giới hạn

Cho dãy vô hạn các vật thể trong không gian 3 chiều $B_1;B_2;...$ có thể tích lần lượt là $V_1;V_2;...$ sao cho $B_{n+1}\subset B_{n}$ và $lim_{n\rightarrow +\infty}V_n=0$. Chứng minh rằng các vật thể này luôn có các điểm chung nằm trong một hình phẳng nào đó có diện tích hữu hạn




#696075 Đề thi chọn đội tuyển Amsterdam lần 3

Đã gửi bởi Minhnksc on 04-11-2017 - 21:18 trong Tài liệu - Đề thi

Với cách tô màu như trên của bạn thì B;D cùng màu có khoảng cách là 1 và nó rơi vào trường hợp 1

Mình nhìn nhầm góc trong bài của bạn :)




#696066 Đề thi chọn đội tuyển Amsterdam lần 3

Đã gửi bởi Minhnksc on 04-11-2017 - 20:57 trong Tài liệu - Đề thi

Câu 5

Xét hình thoi ABCD có $\widehat{A}=60^{0}$

Theo Dirichlet tồn tại 2 đỉnh được tô cùng màu

+) cạnh nối hai đỉnh không phải là AC thì bt được cm

+) cạnh nối hai đỉnh là AC

Ta xét tất cả các hình thoi như trên có chung đỉnh A

Ta chia thành 2 trường hợp như trên

+) cạnh mà Hai đỉnh cùng màu không là cạnh đối của góc 120 độ thì ta có đpcm

+) cạnh mà Hai đỉnh cùng màu là cạnh đối của góc 120 độ 

Dựng $(A;AC)$ thì tất cả các điểm trên đường tròn cùng màu

Lấy 2 điểm có khoảng cách là 1 thì ta có đpcm

P/s:Hành văn hơi kém  :D

Cái chỗ tô đỏ có vấn đề rồi. Chẳng hạn xét 2 hình bình hành ABCD và AB'C'D'. Ta tô màu như sau: A tô màu đỏ; B;C';D tô màu xanh; các điểm còn lại tô màu vàng thì rõ ràng khẳng định trên sai. Rõ ràng ta chỉ thấy được đường chéo của các hình bình hành (đối góc $120^0$) nối hai đỉnh cùng màu chứ không biết nó có cùng màu với các đường chéo của các hình bình hành khác hay không




#695934 Thông tin về VMF trên Alexa

Đã gửi bởi Minhnksc on 01-11-2017 - 21:44 trong Diễn đàn Toán học trên chặng đường phát triển

tiếp tục tăng hạng

Untitled12.png




#695825 $f(x,y)+f(y,z)+f(z,x)=f(0,x+y+z)$

Đã gửi bởi Minhnksc on 30-10-2017 - 12:41 trong Phương trình hàm

Tìm các hàm $f:\mathbb{Q}^2\rightarrow \mathbb{Q}$ thỏa mãn

$f(x,y)+f(y,z)+f(z,x)=f(0,x+y+z)$ với mọi $x;y;z\in \mathbb{Q}$




#695417 Chuyện về những người ăn học không đến nơi đến chốn - bb1412 và vth

Đã gửi bởi Minhnksc on 25-10-2017 - 14:28 trong Quán hài hước

Anh có thể miêu tả kĩ hơn độ điêu luyện trong kiếm pháp và sức mạnh của NĐTS được không ạ? Có vẻ NĐTS này võ công cao cường quá




#695329 Thông tin về VMF trên Alexa

Đã gửi bởi Minhnksc on 23-10-2017 - 23:08 trong Diễn đàn Toán học trên chặng đường phát triển

1223!!!!!!!!!!!!!!

Untitled.png




#695317 Đề thi chọn đội tuyển quốc gia tỉnh Nam Định năm 2017- 2018

Đã gửi bởi Minhnksc on 23-10-2017 - 21:51 trong Thi HSG cấp Tỉnh, Thành phố. Olympic 30-4. Đề thi và kiểm tra đội tuyển các cấp.

Bài tổ hợp 

1)(ngày 2)

ta thấy rằng các đoạn $OA_k$ đi qua một điểm $(p;q)$ nguyên khi và chỉ khi $\frac{p}{q}=\frac{k}{100}$ và $q<100$ 

Có nghĩa là khi này phân số $\frac{k}{100}$ không tối giản. 

Do đó số đoạn $OA_k$ thỏa mãn đề bằng số các số nguyên dương $k<100$ và nguyên tố với 100 hay là bằng $\phi(100)=40$

1)(ngày 1)

Gỉa sử tam giác $A_{i} A_{j}A_{k}$ thỏa mãn đề ; khi đó số đo của các cung nhỏ chắn bởi 3 cạnh $A_{i}A_{j}$;$A_{j}A_{k}$:$A_{k}A_{i}$ lần lượt có dạng $\frac{360x}{2017};\frac{360y}{2017};\frac{360z}{2017};$ với $x+y+z=2017(1)$(x;y;z là các số nguyên dương). Giả sử $m\le n$ và $m\le p$

vì tam giác trên nhọn nên $x;y;z<\frac{2017}{2}$ hay $x;y;z\le 1009$. Đặt $m=1009-x;n=1009-y;p=1009-z$. Khi này; (1) trở  thành

$m+n+p=1010(*)$

Nếu $m=0$ thì số nghiệm tự nhiên của $(*)$ (trong đó $m;n$ khác 0) là $504$

Nếu $m\neq 0$ thì số nghiệm nguyên dương của $(*)$ là $C^{2}_{1009}:6=84756$

Khi đó tổng số các bộ số $x;y;z$ thỏa mãn đề là 85260

Tuy nhiên mỗi tam giác có một bộ 3 cung chắn như trên đều có thể quay 2017 lần để tạo ra các hình tam giác khác

Vậy số hình tam giác thỏa mãn đề là $171969420$ :wacko:




#694661 Tập hợp đề thi Toán các tỉnh thành qua các năm (Update 2017-2018)

Đã gửi bởi Minhnksc on 12-10-2017 - 23:36 trong Thi HSG cấp Tỉnh, Thành phố. Olympic 30-4. Đề thi và kiểm tra đội tuyển các cấp.

Năm 2017-2018

 

$\mathbf{\boxed{I.}}$Chọn đội tuyển thi VMO 

$\mathbf{\boxed{1}}$ THPT Chuyên KHTN Vòng 1 (Ngày 1+2)

$\mathbf{\boxed{2}}$ THPT Chuyên KHTN Vòng 2 (Ngày 3+4)

$\mathbf{\boxed{3}}$ Thành Phố Đà Nẵng

$\mathbf{\boxed{4}}$ Tỉnh Đắc Lắc

$\mathbf{\boxed{5}}$ Tỉnh Cà Mau

$\mathbf{\boxed{6}}$ Tỉnh Bà Rịa Vũng Tàu 

$\mathbf{\boxed{7}}$ Quốc Học Huế

$\mathbf{\boxed{8}}$ Tỉnh Quảng Ninh

$\mathbf{\boxed{9}}$ Tỉnh Hải Dương

$\mathbf{\boxed{10}}$ Thành phố Hà Nội Vòng 2

$\mathbf{\boxed{11}}$ Tỉnh Thái Nguyên

$\mathbf{\boxed{12}}$ Tỉnh Phú Yên

$\mathbf{\boxed{13}}$ THPT Chuyên Phan Bội Châu 

$\mathbf{\boxed{14}}$ Tỉnh Hòa Bình

$\mathbf{\boxed{15}}$ PTNK TP Hồ Chí Minh

          $\mathbf{\boxed{16}}$ Tỉnh Nam Định

          $\mathbf{\boxed{17}}$ Tỉnh Đồng Nai

          $\mathbf{\boxed{18}}$ Tỉnh Cà Mau

          $\mathbf{\boxed{19}}$ Tỉnh Quảng Ninh

          $\mathbf{\boxed{20}}$ Tỉnh Bà Rịa-Vũng Tàu

          $\mathbf{\boxed{21}}$ Tỉnh Thanh Hóa

 

II. Olympic 30/4, GGTH

1. Olympic 30/4 năm 2017 khối 11

Olympic 30/4 năm 2017 khối 10

2. Trại hè Hùng Vương

Trại hè Hùng Vương khối 11

3. Gặp gỡ Toán học khối 10

Gặp gỡ Toán học khối 11

Gặp gỡ Toán học khối 12

4. Olympic chuyên KHTN

To be continued...

P/s: Anh em ĐHV OLP nào muốn tổng hợp thêm thì cứ việc nhé

@vietnaminmyheart: ĐHV OLP tổng hợp theo form trên nhé

 

Từ Zaraki: Đặt chú ý topic của đề thi (không phải topic này) sẽ đưa đề lên trang chủ, giúp tăng sự chú ý. Chú ý là topic ở trang chủ được sắp thứ tự theo thời gian được lập từ mới nhất đến cũ nhất. Do đó topic đề thi nào mà được tạo lâu rồi sẽ không thấy ở trang đầu của trang chủ. Cho nên chỉ đưa chú ý topic đề thi nào mà mới lập + đồng thời bỏ chú ý topic đề thi cũ. Nếu cái này không hoạt động thì PM BQT. ĐHV Olympic lúc nào rảnh thì update đề nhé.  :)




#693749 $(pq)^x|x^n-1$

Đã gửi bởi Minhnksc on 26-09-2017 - 19:09 trong Số học

Cho số nguyên $x$ và hai số nguyên tố lẻ $p;q$ sao cho $x;p;q$ đôi một nguyên tố cùng nhau và $r;s$ lần lượt là hai ước nguyên tố lớn nhất của $p-1;q-1$.Tìm số $n$ nhỏ nhất sao cho $(pq)^x|x^n-1$

P/S




#693701 Ảnh thành viên

Đã gửi bởi Minhnksc on 25-09-2017 - 20:52 trong Góc giao lưu

Khuôn mặt khắm lọ =)). Chủ yếu là khoe cây nhà lá vườn và cho ae biết khuôn mặt của mình nó ntn

21013363_754330761426463_49428434_n.jpg




#693477 tài liệu

Đã gửi bởi Minhnksc on 21-09-2017 - 20:19 trong Tài liệu, chuyên đề, phương pháp về Tổ hợp và rời rạc

Ở đây: http://math.tut.fi/~.../GT_English.pdf

Hoặc ra mua cuốn TLCT 12 có một chương viết về Graph




#692995 VÌ SAO PHÂN SỐ $\frac{1}{0}$ KHÔNG TỒN TẠI ?

Đã gửi bởi Minhnksc on 13-09-2017 - 20:38 trong Toán học lý thú

:D 0!=1 là qui ước bạn ạ,mà bằng cách qui ước đó (  hay những qui ước nói chung ) giúp ta có lợi trong một số tính toán .công thức n! bạn viết là hệ quả của cách định nghĩa n! cho các số tự nhiên >=1,khi bạn qui ước 0! =1 thì bạn thấy rằng: công thức trên không chỉ đúng với những n>=1 mà tại  n =0 (với qui ước 0!=1)  làm cho công thức n! như bạn viết ở trên đúng cho cả trường hợp n=0....cái ''đúng'' này là nhờ bạn quy ước mà có,nên bạn lại ko thể lấy cái đúng này để chứng minh cho cái sinh ra tính đúng của nó.

Mình nghĩ là không nên định nghĩa $n!$ là "tích n số tự nhiên đầu tiên" vì nếu $n=0$ thì làm sao mà tính được kiểu "tích 0 số tự nhiên đầu tiên".Theo mình; $n!$ nên được định nghĩa là các cách sắp xếp $n$ phần tử trong tập hợp có $n$ phần tử. Như thế thì với $n\geq 1$; số $n!$ vẫn thỏa mãn công thức $n!=n(n-1)...2.1$; còn với $n=0$ thì số cách sắp xếp các phần tử trong tập hợp rỗng là 1 cách (để nguyên nó) nên $0!=1$

Nhưng mà khổ nỗi là nếu định nghĩa $n!$ là "tích n số tự nhiên đầu tiên" thì ta mới không biết tính $0!$ kiểu gì và mới sinh ra cái gọi là "quy ước" chứ :)).




#692974 $\boxed{\text{TOPIC}}$ Ôn thi học si...

Đã gửi bởi Minhnksc on 13-09-2017 - 12:47 trong Tài liệu - Đề thi

Gọi là "anh" được rồi nhé :D

Anh đi ngang qua đây vì thấy bài toán của Minhksnc, theo anh thì không hợp với học sinh giỏi lớp 9. Lượng giác ở cấp hai chỉ dừng lại ở khái niệm làm quen qua hình học, chứ chưa đá động tới radiant. Vậy nên anh đang cần lời giải thích của Minksnc cho bài toán kia.

Cảm ơn anh; em hơi thiếu sót một tý. Bài toán số 15 đã được sửa đề để phù hợp hơn với THCS

P/S




#692869 $\boxed{\text{TOPIC}}$ Ôn thi học si...

Đã gửi bởi Minhnksc on 11-09-2017 - 21:06 trong Tài liệu - Đề thi

Bài toán 15: Cho $a,b,c$ là 3 số thực dương thỏa $a+b+c\geq abc$. Chứng minh rằng ít nhất 2 trong 3 bất đảng thức sau đúng:

$\frac{2}{a}+\frac{3}{b}+\frac{6}{c}\geq 6;\frac{6}{a}+\frac{2}{b}+\frac{3}{c}\geq 6;\frac{3}{a}+\frac{6}{b}+\frac{2}{c}\geq 6$




#692658 Bài toán số 3 USA December TST 2016

Đã gửi bởi Minhnksc on 08-09-2017 - 22:58 trong Đa thức

Cho hai đa thức $P;Q \in \mathbb{R}\left[x\right]$ nguyên tố cùng nhau. Chứng minh rằng không có quá 3 số thực $\lambda$ thỏa mãn $P+\lambda Q$ là bình phương của một đa thức




#692566 ĐỀ THI CHỌN ĐT HSG QG VÒNG 1 TỈNH ĐẮC NÔNG NĂM 2018

Đã gửi bởi Minhnksc on 07-09-2017 - 20:27 trong Thi HSG cấp Tỉnh, Thành phố. Olympic 30-4. Đề thi và kiểm tra đội tuyển các cấp.

Câu 1:

Phương trình ban đầu tương đương:

$\frac{1+\sqrt{1+(\frac{1}{x})^2}}{1+\sqrt{1+(f(x))^2}}=\frac{f(x)}{\frac{1}{x}}\Leftrightarrow \frac{1}{x}(1+\sqrt{1+(\frac{1}{x})^2})=f(x)(1+\sqrt{1+(f(x))^2})$

Ta nhận thấy rằng hàm $g(t)=t(1+\sqrt{1+t^2})$ đồng biến trên $\mathbb{R}$ nên chỉ có duy nhất $f(x)=\frac{1}{x}$ làm thỏa mãn phương trình ban đầu.




#692561 ĐỀ THI CHỌN ĐT HSG QG VÒNG 1 TỈNH ĐẮC NÔNG NĂM 2018

Đã gửi bởi Minhnksc on 07-09-2017 - 20:15 trong Thi HSG cấp Tỉnh, Thành phố. Olympic 30-4. Đề thi và kiểm tra đội tuyển các cấp.

Câu 2:

2) Vì $\hat{A}=\pi-\hat{B}-\hat{C}$ và $\hat{B}\leq \frac{\pi}{2}$ nên $\hat {A}\geq \frac{\pi}{2}-\hat{C}$

Do đó $(sinA+sinC)^2\geq (sin(\frac{\pi}{2}-\hat{C})+sinC)^2=sin^2(\frac{\pi}{2}-\hat{C})+sin^2C+2sin(\frac{\pi}{2}-\hat{C})sinC=1+2sin(\frac{\pi}{2}-\hat{C})sinC> sin^2B \Rightarrow sinA+sinC>sinB $

Ta lại có nhận xét sau: Nếu $b>a$ thì $\frac{a}{b}<\frac{a+m}{b+m}$ với $m;a;b$ là các số nguyên dương nên

$\frac{sinB}{sinA+sinB+sinC}<\frac{sinB}{sinA+sinC}<\frac{sinB+sinB}{sinB+sinA+sinC}=\frac{2sinB}{sinA+sinB+sinC}(1)$

Tương tự $\frac{sinA}{sinA+sinB+sinC}<\frac{sinA}{sinB+sinC}<\frac{2sinA}{sinA+sinB+sinC}(2)$

                $\frac{sinC}{sinA+sinB+sinC}<\frac{sinC}{sinB+sinA}<\frac{2sinC}{sinA+sinB+sinC}(3)$

Cộng 3 vế $(1);(2);(3)$ ta có $1<M<2$ nên $\left[M\right]=1$